3
$\begingroup$

I have the following sum $\sum_{j=1}^K {K \choose j} (-1)^{j+1}/j$. Now I can write this as the integral $\int_{-1}^0 \frac{(1+x)^K - 1}{x} dx$. However, I wonder whether there is a closed form expression for that integral? Thanks.

$\endgroup$
1
  • 2
    $\begingroup$ Let $u=x+1$. $\int_{-1}^0 \frac{(1+x)^K-1}{x}dx = \int_0^1 \frac{u^K-1}{u-1}du = \int_0^1 (u^{K-1} + u^{K-2} + ... + u + 1) du$. The result is the $K$th harmonic number $H_K = 1/K + 1/(K-1) + ... + 1/2 + 1$. $\endgroup$ Jan 19, 2014 at 6:36

2 Answers 2

6
$\begingroup$

The closed form of the integral is

$$ \int_{-1}^{0}\frac{(1+x)^k - 1}{x} = \frac{s(k+1,2)}{k!} $$

where $s(k+1,2)$ denote the Stirling number of the first kind.

$\endgroup$
6
  • $\begingroup$ Is there a similar integral representation for $s(n,r)$ ? $\endgroup$ Feb 4, 2013 at 12:32
  • 1
    $\begingroup$ Are you sure about this? Then this would just reduce to $\sum_{j=1}^k 1/k$, right? $\endgroup$
    – Danne
    Feb 4, 2013 at 14:20
  • $\begingroup$ I think one has to involve Stirling functions of the second kind, not Stirling numbers of the first kind. $\endgroup$
    – Danne
    Feb 4, 2013 at 14:36
  • $\begingroup$ @Pietro Yes there is such an integral which is in fact a contour integral. $$ s(n,r) = \frac{n!}{2\pi r!}\int_{|z|=1} z^{-n-1} \log^r (z+1)dz $$ @Danne, Yes I am sure. $\endgroup$ Feb 4, 2013 at 16:10
  • $\begingroup$ But when I evalute that integral and the Stirling formula you gave, I don't get the same answer. How do you find that formula? $\endgroup$
    – Danne
    Feb 4, 2013 at 16:15
2
$\begingroup$

$\newcommand{\+}{^{\dagger}}% \newcommand{\angles}[1]{\left\langle #1 \right\rangle}% \newcommand{\braces}[1]{\left\lbrace #1 \right\rbrace}% \newcommand{\bracks}[1]{\left\lbrack #1 \right\rbrack}% \newcommand{\ceil}[1]{\,\left\lceil #1 \right\rceil\,}% \newcommand{\dd}{{\rm d}}% \newcommand{\down}{\downarrow}% \newcommand{\ds}[1]{\displaystyle{#1}}% \newcommand{\equalby}[1]{{#1 \atop {= \atop \vphantom{\huge A}}}}% \newcommand{\expo}[1]{\,{\rm e}^{#1}\,}% \newcommand{\fermi}{\,{\rm f}}% \newcommand{\floor}[1]{\,\left\lfloor #1 \right\rfloor\,}% \newcommand{\half}{{1 \over 2}}% \newcommand{\ic}{{\rm i}}% \newcommand{\iff}{\Longleftrightarrow} \newcommand{\imp}{\Longrightarrow}% \newcommand{\isdiv}{\,\left.\right\vert\,}% \newcommand{\ket}[1]{\left\vert #1\right\rangle}% \newcommand{\ol}[1]{\overline{#1}}% \newcommand{\pars}[1]{\left( #1 \right)}% \newcommand{\partiald}[3][]{\frac{\partial^{#1} #2}{\partial #3^{#1}}} \newcommand{\pp}{{\cal P}}% \newcommand{\root}[2][]{\,\sqrt[#1]{\,#2\,}\,}% \newcommand{\sech}{\,{\rm sech}}% \newcommand{\sgn}{\,{\rm sgn}}% \newcommand{\totald}[3][]{\frac{{\rm d}^{#1} #2}{{\rm d} #3^{#1}}} \newcommand{\ul}[1]{\underline{#1}}% \newcommand{\verts}[1]{\left\vert\, #1 \,\right\vert}$ \begin{align} &\color{#0000ff}{\large\int_{-1}^{0}{\pars{1 + x}^{K} - 1 \over x}\,\dd x} =\int_{0}^{1}{1 - x^{K} \over 1 - x}\,\dd x =-\int_{x = 0}^{x = 1}\pars{1 - x^{K}}\,\dd\ln\pars{1 - x} \\[3mm]&=\int_{0}^{1}\ln\pars{1 - x}\pars{-Kx^{K - 1}}\,\dd x =-K\lim_{\mu \to 0}\partiald{}{\mu}\int_{0}^{1}\pars{1 - x}^{\mu}x^{K - 1}\,\dd x \\[3mm]&= -K\lim_{\mu \to 0}\partiald{{\rm B}\pars{1 + \mu,K}}{\mu} = -K\lim_{\mu \to 0}\partiald{}{\mu} \bracks{\Gamma\pars{1 + \mu}\Gamma\pars{K} \over \Gamma\pars{1 + \mu + K}} \\[3mm]&=-K\,\Gamma\pars{K}\lim_{\mu \to 0}\bracks{% {\Gamma\pars{1 + \mu}\Psi\pars{1 + \mu} \over \Gamma\pars{1 + \mu + K}} - {\Gamma\pars{1 + \mu}\Psi\pars{1 + \mu + K} \over \Gamma\pars{1 + \mu + K}}} \\[3mm]&= -\Gamma\pars{1 + K}\bracks{% {-\gamma \over \Gamma\pars{1 + K}} - {\Psi\pars{1 + K} \over \Gamma\pars{1 + K}}} = \color{#0000ff}{\large\gamma + \Psi\pars{1 + K}} \end{align}

$$ \begin{array}{rcl} {\rm B}\pars{\alpha,\beta}=\int_{0}^{1}t^{\alpha - 1}\pars{1 - t}^{\beta - 1}\,\dd t: && Beta\ \mbox{function} \\[2mm] \Gamma\pars{z}:&& Gamma\ \mbox{function} \\[2mm] \Psi\pars{z} \equiv \totald{\ln\pars{\Gamma\pars{z}}}{z}:&& Digamma\ \mbox{function} \\[2mm]\gamma \approx 0.577216:&& Euler-Mascheroni\ constant \end{array} $$
$\endgroup$

Your Answer

By clicking “Post Your Answer”, you agree to our terms of service and acknowledge you have read our privacy policy.

Not the answer you're looking for? Browse other questions tagged or ask your own question.